Đến nội dung

the unknown nội dung

Có 210 mục bởi the unknown (Tìm giới hạn từ 20-05-2020)



Sắp theo                Sắp xếp  

#639381 Topic về phương trình và hệ phương trình

Đã gửi bởi the unknown on 10-06-2016 - 17:09 trong Phương trình - hệ phương trình - bất phương trình

Bài 457: Giải hệ phương trình:

          $\left\{\begin{matrix} x=y(4-y)\\ y=z(4-z)\\ z=x(4-x)\\ \end{matrix}\right.$




#635711 Topic về phương trình và hệ phương trình

Đã gửi bởi the unknown on 26-05-2016 - 17:54 trong Phương trình - hệ phương trình - bất phương trình

 

Bài 444: 1)$x^2-x+1-\sqrt{2x-1}=0 $

2)$\frac{3x}{\sqrt{x-1}}=4+\frac{x}{\sqrt{x^2-3x+3}} $

3)$x^4+x^2+6x+9=(x^3+x^2+3x)\sqrt{x+3} $

4)$\frac{1}{\sqrt{-x^2+x+1}}+\frac{1}{\sqrt{-x^2-x+1}}=\frac{2}{\sqrt{1-x^2}}$

 

1.( Câu này chắc là dễ nhất  :) ) .ĐK: $x\geq \frac{1}{2}$

PT $\Leftrightarrow (2x-1)^2+3=4\sqrt{2x-1}$. Đặt $t= \sqrt{2x-1}\Leftrightarrow t^4-4t+3=0\Leftrightarrow (t-1)^2(t^2+2t+3)=0\Leftrightarrow t=1\Leftrightarrow x=1$.

3. ĐK: nếu $x<0$ thì $VT<0<VP$ nên pt vô nghiệm. Vậy $x\geq 0$. Do $x+3\geq 0$ nên áp dụng bất đẳng thức $Cauchy$ ta có:

$x^4+x^2+6x+9=x^4+(x+3)^2\geq \frac{(x^2+x+3)^2}{2}\geq x\sqrt{x+3}(x^2+x+3)\Rightarrow x^2=x+3\Rightarrow x=\frac{1+\sqrt{13}}{2}$ (do $x\geq 0$ nên ta loại nghiệm $ x=\frac{1-\sqrt{13}}{2}$).

4. ĐK: $\frac{1-\sqrt{5}}{2}\leq x\leq \frac{-1+\sqrt{5}}{2}$.

Đặt $\sqrt{-x^2+x+1}=a,\sqrt{-x^2-x+1}=b$ ( $a,b\geq 0$ ). PT $\Leftrightarrow \frac{1}{a}+\frac{1}{b}= \frac{2\sqrt{2}}{\sqrt{a^2+b^2}}\Leftrightarrow (a^2+b^2)(a+b)^2=8a^2b^2$.

Áp dụng BĐT $Cauchy$ có $(a^2+b^2)(a+b)^2\geq 2ab.4ab=8a^2b^2\Rightarrow a=b\Rightarrow x=0$.




#630983 Topic về Bất đẳng thức, cực trị THCS

Đã gửi bởi the unknown on 03-05-2016 - 16:06 trong Bất đẳng thức và cực trị

Bài 4: Chứng minh rằng với mọi số thực dương $a;b;c$ ta có:

             $\sum \frac{1}{a+b}\geq \sum \frac{a}{a^2+bc}$

 

Do tính đối xứng của $a,b,c$ nên ta giả sử $0\leq a\leq b\leq c$

Ta có $VT-VP=\sum \frac{(a-b)(a-c)}{a^2+bc}=\frac{(a-b)^2(a+b)(2c-a-b)}{(a^2+bc)(b^2+ac)}+\frac{(c-a)(c-b)}{c^2+ab}\geq 0$ nên ta có dpcm.




#631019 Topic về Bất đẳng thức, cực trị THCS

Đã gửi bởi the unknown on 03-05-2016 - 19:59 trong Bất đẳng thức và cực trị

Góp vui một câu  :D:

Cho $a;b;c$ là số đo ba cạnh của một tam giác có chu vi là $2$. Tìm giá trị nhỏ nhất của biểu thức $a^2+b^2+c^2+2abc$




#630982 Topic về Bất đẳng thức, cực trị THCS

Đã gửi bởi the unknown on 03-05-2016 - 16:00 trong Bất đẳng thức và cực trị

Bài 1: Cho a,b,c là các số thực dương thỏa mãn $\frac{1}{a+b+1}+\frac{1}{b+c+1}+\frac{1}{a+c+1}\geq 1$.Cmr:

$a+b+c\geq ab+bc+ca$

 

Giả thiết tương đương: $\sum \frac{a+b}{a+b+1}\leq 2\Leftrightarrow 2\geq \sum \frac{(a+b)^2}{(a+b)^2+a+b}\geq \frac{4(a+b+c)^2}{2(a+b+c)+2\sum (a^2+bc)}$

$\Rightarrow a+b+c+(a^2+b^2+c^2+ab+bc+ca)\geq (a+b+c)^2\Leftrightarrow a+b+c\geq ab+bc+ca$




#630969 Topic về Bất đẳng thức, cực trị THCS

Đã gửi bởi the unknown on 03-05-2016 - 12:54 trong Bất đẳng thức và cực trị

Mình cùng góp ý một bài:

Cho a,b,c là các số thực dương thỏa mãn a+b+c=1.Cmr:

$\sum \sqrt{a+b}\geq \frac{3\sqrt{3}}{2\sqrt{2}}.(1+\sum ab)$

Bổ đề: Cho $x,y,z$ dương thỏa $x+y+z=3$, khi đó:$\sqrt{x}+\sqrt{y}+\sqrt{z}\geq xy+yz+zx$

Chứng minh: Ta có $\sqrt{x}+\sqrt{x}+x^2\geq 3x$

Tương tự có: $\sqrt{y}+\sqrt{y}+y^2\geq 3y$

                     $\sqrt{z}+\sqrt{z}+z^2\geq 3z$

Nên $2\sum \sqrt{x}+\sum x^2\geq 3(x+y+z)=9=(x+y+z)^2$$\Rightarrow \sum \sqrt{x}\geq \sum xy$

Áp dụng ta có: $\sum \frac{3}{2}(a+b)= 3\Rightarrow \sqrt{\frac{3}{2}}\sum \sqrt{a+b}\geq \frac{9}{8}\sum (a+b)(b+c)$

Mà $\sum (a+b)(b+c)=(a+b+c)^2+ab+bc+ca=1+\sum ab$

Nên $\sum \sqrt{a+b}\geq \frac{3\sqrt{3}}{2\sqrt{2}}(1+\sum ab)$ $\Rightarrow Q.E.D$




#631560 $\boxed{\text{Chuyên Đề}}$ Bất đẳng thức - Cực trị

Đã gửi bởi the unknown on 06-05-2016 - 12:12 trong Bất đẳng thức và cực trị

Chứng minh:

 

$\frac{a}{1 + b^{2}} + \frac{b}{1+c^{2}} + \frac{c}{1+a^{2}} + \frac{1}{2}\left ( ab + bc + ca \right ) \geq 3$

 

Biết:

 

$a,b,c > 0$ và $\sum \frac{1}{a} \leq 3$

Từ giả thiết $\sum \frac{1}{a} \leq 3$ suy ra $\frac{9}{a+b+c}\leq 3\Rightarrow a+b+c\geq 3$.

Có $\sum \frac{a}{1+b^2}= a+b+c-\sum \frac{ab^2}{1+b^2}\geq a+b+c-\frac{ab+bc+ca}{2}$.

Nên $\sum \frac{a}{1+b^2}+\frac{ab+bc+ca}{2}\geq a+b+c\geq 3$.

Đẳng thức xảy ra khi $a=b=c=3$




#620189 $\boxed{{Topic}}$ Ôn thi học sinh giỏi lớp...

Đã gửi bởi the unknown on 14-03-2016 - 12:14 trong Chuyên đề toán THCS

     Cho $\Delta ABC$ nhọn có trực tâm $H$ và hai đường cao $AD,BE$. Gọi $M,N$ tương ứng là trung điểm hai đoạn $AB,DE$. Đường thẳng $CM$ cắt đường tròn ngoại tiếp tam giác $CDE$ tại điểm $P$ khác $C$. Đường thẳng $CN$ cắt đường tròn ngoại tiếp tam giác $ABC$ tại điểm $Q$ khác $C$.

    Tìm trung trực của $PQ$.

                     ~ Trích đề thi học sinh giỏi tỉnh Đồng Nai ~ :D  :D




#619873 Topic tổng hợp các bài toán về phương trình nghiệm nguyên.

Đã gửi bởi the unknown on 12-03-2016 - 14:20 trong Số học

Tìm nghiệm nguyên dương của phương trình: $a^2+b^2+c^2=2(ab+bc+ca)$




#632446 TOPIC luyện thi vào lớp 10 chuyên toán năm 2016 - 2017.

Đã gửi bởi the unknown on 11-05-2016 - 12:21 trong Tài liệu - Đề thi

Bài BĐT:

Cho ba số dương $x,y,z$. Chứng minh rằng: $$\frac{x}{2x+y+z}+\frac{y}{x+2y+z}+\frac{z}{x+y+2z}\leq \frac{3}{4}.$$

Điều phải chứng minh tương đương $\sum \frac{x+y}{x+y+2z}\geq \frac{3}{2}$.

Mặt khác áp dụng Svac-xơ ta có $\sum \frac{x+y}{x+y+2z}= \sum \frac{(x+y)^2}{(x+y)(x+y+2z)}\geq \frac{4(x+y+z)^2}{\sum (x+y)(x+y+2z)}$.

Do đó ta chỉ cần chứng minh $8(x+y+z)^2\geq 3\sum (x+y)(x+y+2z)\Leftrightarrow \sum (x-y)^2\geq 0$ ( luôn đúng), do đó ta có đpcm.

Đẳng thức xảy ra khi $x=y=z$.




#636644 TOPIC luyện thi vào lớp 10 chuyên toán năm 2016 - 2017.

Đã gửi bởi the unknown on 29-05-2016 - 21:38 trong Tài liệu - Đề thi

Xin lỗi mình nhầm

A = $a+\frac{1}{a}$ =$\frac{a^2+1}{a}$

Vì A là số nguyên dương nên a >0 và $a^2+1 = ak$ (với k là số nguyên dương)

Xét pt bậc 2 $a^2-ak+1=0$

$\Delta=k^2-4$

Pt có nghiệm nên $k\geq2$ hoặc $k\leq-2$

$ a1 = \frac{k+ \sqrt{k^2-4}}{2}  $

$ a2 = \frac{k- \sqrt{k^2-4}}{2}  $

Đồng thời 2 nghiệm của pt là số dương nên

S>0 và P>0

=> k>0

Từ đó suy ra $k\geq2$

Sau đó xét từng giá trị a1 ,a2 sao cho không lớn hơn 2017

suy ra được $2\leq k \leq2017$

Như vậy có 2016 giá trị của k và 4032 giá trị của a thoả mãn đề bài

Mình nghĩ mình có sai sót chỗ nào đó rồi các bạn xem mình làm có đúng không

Một lưu ý cho các bạn là để ý rằng phương trình $a+\frac{1}{a}=2$ chỉ có một nghiệm duy nhất. Do đó số giá trị cần tìm chỉ là $4031$ thôi bạn nhé.




#636469 TOPIC luyện thi vào lớp 10 chuyên toán năm 2016 - 2017.

Đã gửi bởi the unknown on 29-05-2016 - 10:28 trong Tài liệu - Đề thi

Bài 1:

        a) Tìm các cặp số thực $(a,b)$ để phương trình $x^2+ax+b=0$ có hai nghiệm nguyên và $3a+b=8$. Với các cặp số thực $(a,b)$ tìm được, giải phương trình đã cho.

        b) Giải phương trình: $(6x-3)\sqrt{7-3x}+(15-6x)\sqrt{3x-2}=2\sqrt{-9x^2+27x-14}+11$

Bài 2:

        a) Tìm tất cả các cặp số nguyên dương $(x,y,z)$ để $3^{x}+5^{y}=z^3$.

        b) Chứng minh rằng phương trình: 

                                  $\frac{1}{x}+\frac{1}{y}+\frac{1}{z}=\frac{1}{2017}$

hữu hạn nghiệm nguyên dương.

        c) Có bao nhiêu số thực $a$ sao cho biểu thức $a+\frac{1} {a}$ là một số nguyên dương không lớn hơn $2017$.

Bài 3: Chứng minh bất đẳng thức với ba số thực dương $a,b,c$ thỏa $a+b+c=3$:

                                   $a^2b+b^2c+c^2a\geq \frac{9a^2b^2c^2}{1+2a^2b^2c^2}$

Bài 4: Cho tam giác $ABC$ có ba góc nhọn nội tiếp đường tròn $(O)$, $BC$ cố định, $A$ thay đổi trên đường tròn, $BE$ và $CF$ là các đường cao. Các tiếp tuyến với đường tròn $(O)$ tại $B$ và $C$ cắt nhau tại $S$, các đường thẳng $BC$ và $OS$ cắt nhau tại $M$.

         a) Chứng minh: $\frac{AB}{AE}=\frac{BS}{ME}$

         b) Giả sử $AS$ cắt $(O)$ tại điểm thứ hai $K$. Chứng minh rằng $MA.MK$ không đổi khi $A$ thay đổi trên đường tròn $(O)$.

Bài 5: Chứng minh rằng với hai số thực không âm $a$ và $b$, ta có bất đẳng thức:

                                        $[2a]+[2b]\geq [a]+[b]+[a+b]$

trong đó $[a]$ là kí hiệu của số nguyên lớn nhất không vượt quá a.

Spoiler




#631758 TOPIC luyện thi vào lớp 10 chuyên toán năm 2016 - 2017.

Đã gửi bởi the unknown on 07-05-2016 - 16:24 trong Tài liệu - Đề thi

Kết quả thì ra đúng rồi, nhưng bạn xem lại phần chữ đỏ nha, không ổn cho lắm

Sao lại không ổn vậy bạn?




#631456 TOPIC luyện thi vào lớp 10 chuyên toán năm 2016 - 2017.

Đã gửi bởi the unknown on 05-05-2016 - 21:01 trong Tài liệu - Đề thi

Góp một bài dễ thôi: Cho $a,b,c$ là các số thực dương thỏa $ab+bc+ca=3$. Chứng minh rằng:

                $\frac{1}{a^2+1}+\frac{1}{b^2+1}+\frac{1}{c^2+1}\geq \frac{3}{2}$




#631727 TOPIC luyện thi vào lớp 10 chuyên toán năm 2016 - 2017.

Đã gửi bởi the unknown on 07-05-2016 - 12:52 trong Tài liệu - Đề thi

Góp một bài khá là hay nha:
Chứng minh rằng với mọi $n\geq 6$ thì $a_{n}$ luôn là số chính phương với $a_{n}=1 + $ $\frac{2.6.10.....(4n-2)}{(n+5)(n+6)...(2n)}$

Có $\frac{2.6.10.....(4n-2)}{(n+5)(n+6)...(2n)}=2^n.\frac{1.3.5...(2n-1)}{(n+5)(n+6)...(2n)}=\frac{1.2.3....(2n-1)(2n)}{1.2.3...n.(n+5)(n+6)...(2n)}= (n+1)(n+2)(n+3)(n+4)$.

Vậy $a_{n}=1+(n+1)(n+2)(n+3)(n+4)=(n^2+5n+5)^2$ là một số chính phương.




#631461 TOPIC luyện thi vào lớp 10 chuyên toán năm 2016 - 2017.

Đã gửi bởi the unknown on 05-05-2016 - 21:06 trong Tài liệu - Đề thi

Tiện đây "cống hiến" cho topic một bài toán luôn cho đỡ bị loãng topic:

Tìm x,y nguyên dương thoả mãn phương trình: $25x+46=y(y+1)$

Giả thiết tương đương $100x+185=(2y+1)^2$. Do đó $(2y+1)^2$ là số chính phương lẻ chia hết cho 5 nên $(2y+1)^2\equiv 25$ ( $mod$ $100$).

Mà $100x+185\equiv 85$ ( $mod$ $100$) nên vô lý.

Vậy không tồn tại $x,y$ thỏa mãn đề bài.




#631474 TOPIC luyện thi vào lớp 10 chuyên toán năm 2016 - 2017.

Đã gửi bởi the unknown on 05-05-2016 - 21:22 trong Tài liệu - Đề thi

Mở rộng bài toán ra tập hợp số thực thì có giải được không bạn?

Dĩ nhiên là được chứ bạn. Điều này cũng đúng với tập hợp số hữu tỉ vì khi đó $x=\frac{y^2+y-46}{25}$ nên nếu $y$ là số hữu tỉ thì $x$ cũng là số hữu tỉ.




#631469 TOPIC luyện thi vào lớp 10 chuyên toán năm 2016 - 2017.

Đã gửi bởi the unknown on 05-05-2016 - 21:16 trong Tài liệu - Đề thi

Thêm một bài nữa cho vui: Giả sử $m,n,p$ ($n$ là hằng số, $m,p$ là các số thực thay đổi) là các số thực sao cho phương trình $x^4+mx^3+nx^2+px+1$ có bốn nghiệm thực đều âm ( không nhất thiết phân biệt). Tìm giá trị lớn nhất của $p$ theo $n$.




#646983 Marathon Phương trình và hệ phương trình VMF

Đã gửi bởi the unknown on 29-07-2016 - 08:45 trong Phương trình - hệ phương trình - bất phương trình

Bạn thử kiểm tra lại.  Chú ý số hạng $\dfrac{x^{2}+x\sqrt{x}+2}{x+x\sqrt{x}+4}.$

 

Phương trình này có ít nhất một nghiệm khác trong $\left(\frac{3}{2},2\right)$?

À vâng em có đã kiểm tra và hơi sai sót một tí. Thật sự ra thì cách giải của em không có vấn đề gì mà do anh NTA1907 ghi đề bị sai sót đó anh. Đề chính xác phải là:

$\dfrac{3+\sqrt{x}}{x^{2}+x\sqrt{x}+x+3}+\dfrac{x+\sqrt{x}+2}{x^{2}+x\sqrt{x}+4}+\dfrac{x\sqrt{x}+x+2}{x^{2}+\sqrt{x}+4}+\dfrac{x^{2}+x\sqrt{x}+2}{x+\sqrt{x}+4}+\dfrac{x^{2}+3}{x\sqrt{x}+x+\sqrt{x}+3}=\dfrac{10}{3}$

Tức là phải sửa số hạng $\frac{x^2+x\sqrt{x}+2}{x+x\sqrt{x}+4}$ thành $\frac{x^2+x\sqrt{x}+2}{x+\sqrt{x}+4}$.

Đây hình như là đề đề nghị Olympic 30/4.

Nhân tiện vì không có ai gửi bài nên em xin tiếp tục:

Bài 93: Giải phương trình: $\sqrt{2x+15}=32x^2+32x-20$.




#646935 Marathon Phương trình và hệ phương trình VMF

Đã gửi bởi the unknown on 28-07-2016 - 20:32 trong Phương trình - hệ phương trình - bất phương trình

Bài 91: Giải phương trình:

$\dfrac{3+\sqrt{x}}{x^{2}+x\sqrt{x}+x+3}+\dfrac{x+\sqrt{x}+2}{x^{2}+x\sqrt{x}+4}+\dfrac{x\sqrt{x}+x+2}{x^{2}+\sqrt{x}+4}+\dfrac{x^{2}+x\sqrt{x}+2}{x+x\sqrt{x}+4}+\dfrac{x^{2}+3}{x\sqrt{x}+x+\sqrt{x}+3}=\dfrac{10}{3}$

Bài này thì ý tưởng là dùng bất đẳng thức:

Điều kiện $x\geq 0$

Ta đặt $a=2,b=\sqrt{x}+1,c=x^2+1,d=x\sqrt{x}+1,e=x+1$. Khi đó phương trình tương đương: $\sum \frac{a+b}{c+d+e}\geq \frac{10}{3}$

Ta sẽ chứng minh: $\sum \frac{a+b}{c+d+e}\geq \frac{10}{3}$ với mọi $a,b,c,d,e$ dương.

Thật vậy ta có: $\sum \frac{a+b}{c+d+e}\geq \frac{10}{3}\Leftrightarrow (a+b+c+d+e)\sum \frac{1}{a+b+c}\geq \frac{25}{3}$.

Mà theo BĐT Cauchy Schwarz thì $\sum \frac{1}{a+b+c}\geq \frac{25}{3(a+b+c+d+e)}$ 

Tù đó ta suy ra điều phải chứng minh. Hơn nữa đẳng thức xảy ra khi và chỉ khi $a=b=c=d=e$ tức là $x=1$. Từ đó suy ra phương trình có nghiệm duy nhất $x=1$.

 

Mạn phép cho em nói một tí  :D : Em thấy topic có vẻ hơi lạc hướng của Marathon mà dần trở thành Topic thảo luận về phương trình và hệ phương trình rồi ạ.

 

Vì hướng tới thi olympic nên em xin đề xuất bài tiếp theo:

Bài 92: Giải hệ phương trình:

$\left\{\begin{matrix} x_1+x_2+x_3+x_4+x_5=1\\ (x_1+x_2)(x_1+x_2+x_3)(x_1+x_2+x_3+x_4)=256x_1x_2x_3x_4x_5\\ x_1,x_2,x_3,x_4,x_5> 0\\ \end{matrix}\right.$




#637028 Marathon Phương trình và hệ phương trình VMF

Đã gửi bởi the unknown on 31-05-2016 - 09:29 trong Phương trình - hệ phương trình - bất phương trình

 

Bài toán 20: $\left\{\begin{matrix} &(x+y)^{2}+3x+2y+4=3\sqrt[3]{4x-4} \\ &2(y+1)^{2}(y+1-\sqrt[3]{18x-x^{3}})=17\left ( \sqrt{x-2} \right )^{2}-35-10x^{2} \end{matrix}\right.$

 

 

Từ phương trình $(2)$ ta suy ra $x\geq 2$ nên $\sqrt[3]{4x-4}> 0$, từ đó áp dụng bất đẳng thức $Cauchy$ ta có: $3.2.2.\sqrt[3]{4x-4}\leq 4x-4+8+8=4x+12\Rightarrow 3\sqrt[3]{4x-4}\leq x+3$

Do đó $(x+y)^2+3x+2y+4\leq x+3\Rightarrow (x+y+1)^2\leq 0$. Mà $(x+y+1)^2\geq 0$ với mọi $x,y$ nên $(x+y+1)^2=0$. Đẳng thức xảy ra khi và chỉ khi:

                        $\left\{\begin{matrix} x+y+1=0\\ 4x-4=8\\ \end{matrix}\right. \Leftrightarrow \left\{\begin{matrix} x=3\\ y=-4\\ \end{matrix}\right.$

Thử lại thấy nghiệm $(x,y)=(3;-4)$ thỏa phương trình $(2)$ nên hệ phương trình có một nghiệm là $(3;-4)$.




#636819 Marathon Phương trình và hệ phương trình VMF

Đã gửi bởi the unknown on 30-05-2016 - 15:07 trong Phương trình - hệ phương trình - bất phương trình

Bài toán 19: Giải hệ phương trình: 

                 $\left\{\begin{matrix} 2x-2y+\sqrt{x+y+3xy+1}=1\\ \sqrt[3]{3y+1}=8x^2-2y-1\\ x>0\\ \end{matrix}\right.$




#636817 Marathon Phương trình và hệ phương trình VMF

Đã gửi bởi the unknown on 30-05-2016 - 14:48 trong Phương trình - hệ phương trình - bất phương trình

 

 

Bài toán 18: Giải phương trình:

$\sqrt{3x^{2}-6x-5}=\sqrt{(2-x)^{5}}+\sqrt{2-x}(2x^{2}-x-10)$

Điều kiện: $(3x^2-5x-6)\sqrt{2-x}= \sqrt{3x^2-6x-5}\Rightarrow \left\{\begin{matrix} x\leq 2\\ 3x^2-5x-6\geq 0\\ 3x^2-6x-5\geq 0\\ \end{matrix}\right. \Leftrightarrow x\leq \frac{5-\sqrt{97}}{6}$

Phương trình tương đương: $\sqrt{3x^2-6x-5}=\sqrt{2-x}(2x^2-x-10+(2-x)^2)=\sqrt{2-x}(3x^2-5x-6)$

Đặt $2-x=a,3x^2-5x-6=b\Rightarrow b\sqrt{a}=\sqrt{b+a-1}\Leftrightarrow ab^2=b+a-1\Leftrightarrow a(b^2-1)=b-1\Leftrightarrow (b-1)(ab+a-1)=0\Rightarrow \begin{bmatrix} b=1\\ ab+a-1=0\\ \end{bmatrix} \Rightarrow \begin{bmatrix} 3x^2-5x-7=0\\ (2-x)(3x^2-5x-6)=x-1\\ \end{bmatrix}$

Nếu $3x^2-5x-7=0$ $\Leftrightarrow x=\frac{5\pm \sqrt{109}}{6}$, mà $x\leq \frac{5-\sqrt{97}}{6}$ nên $x=\frac{5-\sqrt{109}}{6}$.

Nếu $ (2-x)(3x^2-5x-6)=x-1\Rightarrow x\geq 1$ mâu thuẫn với điều kiện $x\leq \frac{5-\sqrt{97}}{6}$ nên phương trình vô nghiệm.

Vậy phương trình có một nghiệm là $x=\frac{5-\sqrt{109}}{6}$.




#636872 Marathon Phương trình và hệ phương trình VMF

Đã gửi bởi the unknown on 30-05-2016 - 18:24 trong Phương trình - hệ phương trình - bất phương trình

 

hình như cái pt 2 là -11x^2 bạn ơi ...... !! :D

Chết thật! Nhầm lẫn nghiêm trọng quá, cám ơn anh đã nhắc  :D . Mà em nghĩ anh nên làm chặt điều kiện hơn nữa ( mặc dù nghiệm vẫn thỏa điều kiện )




#636879 Marathon Phương trình và hệ phương trình VMF

Đã gửi bởi the unknown on 30-05-2016 - 19:00 trong Phương trình - hệ phương trình - bất phương trình

không sao đâu bạn .... mình cũng bị vậy nhiều mà ......... chuyện đó cũng bình thường thôi ......... làm chặt điều kiện hơn nữa là sao bạn ?........... cách 2 của mình làm giống cách bạn nhưng hình như pt bậc 3 có 1 nghiệm thỏa điều kiện ( mình bấm máy . :D :D)  mà cách mình đăng là vô nghiệm ....... các bạn xem lại nha . !! :D

à ý của em là điều kiện có thể làm chặt hơn là $x\leq \frac{5-\sqrt{97}}{6}< \frac{3-2\sqrt{6}}{3}$, và với điều kiện này thì nghiệm của phương trình bậc ba kia không thỏa mãn ( em đã sửa bài và chứng minh phương trình bậc ba đó vô nghiệm bằng điều kiện mới).